0
$\begingroup$

Could somebody explain in detail the steps required to solve this problem?

Let $p$ be a prime dividing $o(G)$. Show that every Sylow $p$-subgroup of $G/K$ is of the form $PK/K$, where $P$ is a Sylow $p$-subgroup of $G$.

$\endgroup$

1 Answer 1

1
$\begingroup$

I'll sketch some steps for you:

(i) Suppose $|K| = p^a m$ and $|G| = p^{a + b} mn$, where $m$ and $n$ are not divisible by $p$. Then $|G / K| = p^b n$. What is the order of a Sylow $p$-subgroup of $G / K$?

(ii) Show that every subgroup of $G / K$ is of the form $H/K$, where $H$ is some subgroup of $G$ containing $K$. Deduce that any Sylow $p$-subgroup of $G / K$ is of the form $H/K$ for some subgroup $H$ of order $p^{a + b}m$.

(iii) By Sylow's theorems, this $H$ contains a Sylow $p$-subgroup $P$. What is the order of $P$? Convince yourself that $P$ is also a Sylow $p$-subgroup of $G$.

(iv) If you can now show that $ PK = H$, then you are done. Since you already know that $P$ and $K$ are both subgroups of $H$, it is enough (why?) to show that $|PK| = |H|$, i.e. that $|PK| = p^{a+b}m$. But $|P| = p^{a + b}$ and $|K|=p^a m$, and $m$ is not divisible by $p$. Now apply Lagrange's theorem...

$\endgroup$
7
  • $\begingroup$ Sorry i am weak in mathematics and i dont understand, please explain steps in clearly .. $\endgroup$
    – Raj
    Oct 26, 2017 at 16:35
  • $\begingroup$ @Raj Could you please tell me what you already understand and what you don't understand? Also, did you make any progress on this question yourself? What are your thoughts? $\endgroup$
    – Kenny Wong
    Oct 26, 2017 at 20:12
  • $\begingroup$ I dont understand anything about this question, this question is a assignment for me, i need a solution for this question..and i have a 20 questions more to solve for assignment i forgot mathematics because from 3 years gap please help out tommarow is the last date for submition and if you help me to answer another questions i will send you that questions also if you dont mind send me your mail is i will send through in that mail $\endgroup$
    – Raj
    Oct 27, 2017 at 2:27
  • 1
    $\begingroup$ Dear @Raj, I'm sorry to hear about your situation, but this website is not an assignment-solving service. However, if you state what have already learnt about Sylow's theorems and what you've tried for this question and where you're stuck, then people will be more than happy to help you out. :) $\endgroup$
    – Kenny Wong
    Oct 27, 2017 at 6:47
  • $\begingroup$ Could you please suggest me some books for learn mathematics in understanding way...for M.Sc $\endgroup$
    – Raj
    Oct 27, 2017 at 8:50

Not the answer you're looking for? Browse other questions tagged .